Jump to content

viet9a14124869's Content

There have been 855 items by viet9a14124869 (Search limited from 26-05-2020)



Sort by                Order  

#671991 Chuyên đề Hệ phương trình

Posted by viet9a14124869 on 18-02-2017 - 17:19 in Chuyên đề toán THPT

bài này khi nhân chéo rồi thực hiện bình phương thì nhận thấy cần cm $c(a-b)(c^2-ab)\geq 0\Leftrightarrow$ $a\geq b$ nên chắc thiếu điều kiện  

mọi người giúp mình câu này ạ: 

cho a,b,c thỏa mãn a,b>0 và c>$\sqrt{ab}$ . chứng minh rằng:

$\frac{a+c}{\sqrt{a^2+c^2}} \geq \frac{b+c}{\sqrt{b^2+c^2}}$

 

giải hệ phương trình:

a, $\left\{\begin{matrix} xy(x^2+y^2)=2 & \\ 2x^5=(x+y)(x^4+y^4+x^2y^2-2) & \end{matrix}\right.$

b, $\left\{\begin{matrix} \sqrt[3]{x+2y}=4-x-y & \\ \sqrt[3]{2x+6} +\sqrt{2y} =2 & \end{matrix}\right.$

c, $\left\{\begin{matrix} x^2(1+y^2)=2 & \\ 1+x^2y+xy=3x^2 & \end{matrix}\right.$

d, $\left\{\begin{matrix} y^2+4xy+y-2x=0 & \\ y^4+8xy^2+4x^2+3y^2=0 & \end{matrix}\right.$

e, $\left\{\begin{matrix} 4x-4y^2=x^2y^2 & \\ 3x^2+y^3=12x-13 & \end{matrix}\right.$

mọi người giúp em với ạ

a, thay $xy(x^2+y^2)=2$ vào phương trình 2 thì được $2x^5=x^5+y^5\Rightarrow x=y$ ...đến đây dễ suy ra nghiệm .

e, Từ phương trình 1 suy ra $y^2=\frac{4x}{x^2+4}\leq 1\Rightarrow -1\leq y\leq 1$

Mà theo phương trình 2 thì $y^3+1=3(x-2)^2\geq 0\Rightarrow y\geq -1$

Do đó y=-1,,,x=2




#674047 Topic về Bất đẳng thức, cực trị THCS

Posted by viet9a14124869 on 12-03-2017 - 13:18 in Bất đẳng thức và cực trị

Cho x,y thỏa mãn

$\left ( x^{2}+y^{2}+1 \right )^{2}+4x^{2}y^{2}-x^{2}-y^{2}=0$

Tìm min, max P=$x^{2}+y^{2}$

Đề sai bạn ạ ,,,khi tách hết ra ta được $x^4+6x^2y^2+y^4+x^2+y^2+1=0$ vô lí




#672777 Topic về Bất đẳng thức, cực trị THCS

Posted by viet9a14124869 on 25-02-2017 - 22:21 in Bất đẳng thức và cực trị

Cho a,b,c>0 thỏa $a+b+c\geq 3$.Tìm giá trị nhỏ nhất của biểu thức S=$\frac{a}{\sqrt{b}}+\frac{b}{\sqrt{c}}+\frac{c}{\sqrt{a}}$

Cho x,y,z>0 thỏa mãn x+y+z=1. Tìm giá trị lớn nhất của biểu thức: P=$\frac{x}{x+1}+\frac{y}{y+1}+\frac{z}{z+1}$

1, Ta có $\frac{a}{\sqrt{b}}+\frac{b}{\sqrt{c}}+\frac{c}{\sqrt{a}}=\frac{a^2}{a\sqrt{b}}+\frac{b^2}{b\sqrt{c}}+\frac{c^2}{c\sqrt{a}}\geq \frac{(a+b+c)^2}{a\sqrt{b}+b\sqrt{c}+c\sqrt{a}}\geq \frac{(a+b+c)^2}{\sqrt{(a+b+c)(ab+bc+ca)}}=\frac{9}{\sqrt{3(ab+bc+ca)}}\geq 3\Leftrightarrow a=b=c=1$

2, Bạn thay x+y+z=1 vào mẫu mỗi phân số rồi dùng cô-si là xong




#675998 Topic về Bất đẳng thức, cực trị THCS

Posted by viet9a14124869 on 02-04-2017 - 15:55 in Bất đẳng thức và cực trị

Cho a,b,c là các số thực dương thỏa mãn abc=1.Chứng minh:

$\frac{a}{(a+1)(b+1)}+\frac{b}{(b+1)(c+1)}+\frac{c}{(c+1)(a+1)}\geq \frac{3}{4}$

Cho ba số dương a,b,c thỏa $a+b+c\leq k$ thì $(1+\frac{1}{a})(1+\frac{1}{b})(1+\frac{1}{c})\geq (1+\frac{3}{k})^3$

Câu đầu đồng quy biểu thức ,,,ta cần chứng minh $ab+bc+ca+a+b+c\geq 6$ đúng theo AM-GM

Câu sau ta dùng holder với AM-GM ta có $\prod (\frac{1}{2}+\frac{1}{2}+\frac{1}{a})\geq (1+\frac{1}{\sqrt[3]{abc}})^3\geq (1+\frac{3}{a+b+c})^3\geq (1+\frac{3}{k})^3$




#676656 Topic về Bất đẳng thức, cực trị THCS

Posted by viet9a14124869 on 08-04-2017 - 21:12 in Bất đẳng thức và cực trị

Cho a,b,c>0 và a^2+b^2+c^2=1.CM $\frac{bc}{a^2+1}+\frac{ca}{b^2+1}+\frac{ab}{c^2+1}\leqslant \frac{3}{4}$

Ta có $\frac{b^2}{a^2+b^2}+\frac{c^2}{c^2+a^2}\geq \frac{(b+c)^2}{2a^2+b^2+c^2}\geq \frac{4bc}{2a^2+b^2+c^2}=\frac{4bc}{a^2+1}$

Làm tương tự rồi cộng 3 vế lại ta có đpcm !

Dấu = xảy ra khi $a=b=c=\frac{1}{\sqrt{3}}$




#677413 Topic về Bất đẳng thức, cực trị THCS

Posted by viet9a14124869 on 14-04-2017 - 20:45 in Bất đẳng thức và cực trị

Nếu sử dụng $AM-GM$ dưới mẫu sẽ không thỏa mãn dấu bằng vì $a=b=c=3$ mà.

Bạn tách ra $a+bc=a+\frac{bc}{3}+\frac{bc}{3}+\frac{bc}{3}$ rồi dùng AM-GM  là OK




#677033 Topic về Bất đẳng thức, cực trị THCS

Posted by viet9a14124869 on 11-04-2017 - 06:06 in Bất đẳng thức và cực trị

cho các số thực $a,b,c$. Chứng minh :

$$\frac{ab}{a+b-c}+\frac{bc}{b+c-a}+\frac{ca}{c+a-b}\ge a+b+c$$

 

Đề ra cho là sai $a,b,c\ge 0$ mới đúng

Mình nghĩ nên có thêm điều kiện a,b,c là 3 cạnh tam giác sẽ đúng hơn ,,bởi nếu a=1,b=2,c=4 thì bất đẳng thức sai




#677410 Topic về Bất đẳng thức, cực trị THCS

Posted by viet9a14124869 on 14-04-2017 - 20:35 in Bất đẳng thức và cực trị

1. Cho 3 số  dưeơng a,b,c. CMR: $(\frac{a}{b}+\frac{b}{c}+\frac{c}{a})^{2}\geq(a+b+c)(\frac{1}{a}+\frac{1}{b}+\frac{1}{c})$

2. Cho các só thực x,y, z thỏa mãn điều kiện: x2 +y+z2=1. Tìm GTLN của A=xy+yz+2xz

3. Cho các số thực a,b,c thuộc đoạn [-2;5] tm: a+2b+3c$\leq$2. Tìm GTLN: a2+2b2+3c2

4. Cho a,b,c>0 tm: $\frac{1}{a}+\frac{1}{b}+\frac{1}{c}$=1. CMR:$\frac{a^{2}}{a+bc}+\frac{b^{2}}{b+ca}+\frac{c^{2}}{c+ab}\geq \frac{a+b+c}{4}$

Bài 2 , Ta có $\left\{\begin{matrix} xy\leq \frac{\sqrt{3}-1}{2}x^2+\frac{1}{2\sqrt{3}-2}y^2 & & \\ yz\leq \frac{\sqrt{3}-1}{2}z^2+\frac{1}{2\sqrt{3}-2}y^2 & & \\ 2xz\leq x^2+z^2 & & \end{matrix}\right.\Rightarrow xy+yz+2zx\leq \frac{\sqrt{3}+1}{2}(x^2+y^2+z^2)=\frac{\sqrt{3}+1}{2}\Leftrightarrow x=z=\frac{\sqrt{3}+1}{2}y\Leftrightarrow .........$

Bài 3 , $-2\leq a\leq 5\Leftrightarrow a^2\leq 3a+10 \Rightarrow ..... \Rightarrow a^2+2b^2+3c^2\leq (3a+10)+2(3b+10)+3(3c+10)=3(a+2b+3c)+60\leq  66\Leftrightarrow a=-2,b=5,c=-2$

Bài 4 , Chắc là dùng $\frac{a^2}{a+bc}=a-\frac{abc}{a+bc}$ rồi dùng AM-GM ở mẫu ^_^




#673447 Hướng dẫn gửi bài trên Diễn đàn

Posted by viet9a14124869 on 04-03-2017 - 21:18 in Hướng dẫn - Trợ giúp - Giải đáp thắc mắc khi sử dụng Diễn đàn

 

không thấy gửi bài mới ở đâu

 

https://diendantoanh...-và-cực-trị/

Giả dụ bạn muốn gửi bài trong box Bất đẳng thức thì bấm vào link trên này ,,,sẽ thấy có ô màu đen ghi gửi bài mới đó




#671833 Topic phương trình, hệ phương trình vô tỉ

Posted by viet9a14124869 on 16-02-2017 - 21:23 in Phương trình, hệ phương trình và bất phương trình

\[ Bài 1 : {x^3} + 2\sqrt {{{(3x - 2)}^3}}  = 3x(3x - 2)\]

 Đặt $\sqrt{3x-2}=a\Rightarrow x^3+b^3+b^3=3xb^2\Leftrightarrow x+2b=0$ hoặc $x^2-2bx+3b^2=0$ loại

Vậy ta sẽ tìm được x do x+2b=0




#671851 Topic phương trình, hệ phương trình vô tỉ

Posted by viet9a14124869 on 17-02-2017 - 06:11 in Phương trình, hệ phương trình và bất phương trình

Sai ! thử thay số vào đi bạn 

Nếu thay số mà không  ra thì chắc là vô nghiệm




#671625 Topic phương trình, hệ phương trình vô tỉ

Posted by viet9a14124869 on 14-02-2017 - 19:51 in Phương trình, hệ phương trình và bất phương trình

\[ Bài 3 : {x^2} = \sqrt {{x^3} - {x^2}}  + \sqrt {{x^2} - x} \]

Xét thấy x=0 là nghiệm của bài toán

Nếu $x\neq 0\Rightarrow x\geq 1\Rightarrow 2x^2=2\sqrt{x^2(x-1)}+2\sqrt{(x^2-x).1}\leq (x^2+x-1)+(x^2-x+1)=2x^2\Leftrightarrow x^2=x-1=1-x\Leftrightarrow 1=x=0$ loại

Vậy x=0




#671787 Topic phương trình, hệ phương trình vô tỉ

Posted by viet9a14124869 on 16-02-2017 - 13:35 in Phương trình, hệ phương trình và bất phương trình

máy mình cũng bị tương tự

giải phương trình 4{x-\sqrt(5-x)}.{\sqrt(5-x) +3}=(x+3)^2

 

bạn nào sửa hộ mih đk máy mih gặp sự cố

Sử dụng bdt cauchy cho 2 số ta có 4ab<=(a+b)^2 ....

Do đó ta cm được VT<=VP 




#671623 Topic phương trình, hệ phương trình vô tỉ

Posted by viet9a14124869 on 14-02-2017 - 19:46 in Phương trình, hệ phương trình và bất phương trình

\[ Bài 2 : \sqrt {4{x^2} + 5x + 1}  + 3 = 2\sqrt {{x^2} - x + 1}  + 9x\]

Ta có $\Leftrightarrow \sqrt{4x^2+5x+1}-\sqrt{4x^2-4x+4}=9x-3\Leftrightarrow \frac{9x-3}{\sqrt{4x^2+5x+1}+\sqrt{4x^2-4x+4}}=9x-3$

Xét $x\neq \frac{1}{3}\Rightarrow 1=\sqrt{4x^2+5x+1}+\sqrt{4x^2-4x+4}\geq \sqrt{3}> 1$ loại

Do đó $x=\frac{1}{3}$




#671667 Topic: Các bài toán về tính chia hết

Posted by viet9a14124869 on 14-02-2017 - 22:30 in Số học

Số dư khi chia $[n^{2}+1]^{2016} cho n$ là bao nhiêu

1 theo đồng dư thức ^-^




#676171 Topic: Các bài toán về tính chia hết

Posted by viet9a14124869 on 04-04-2017 - 12:40 in Số học

- Chứng minh rằng 210+512 là hợp số .
Mod và các bác giúp em với :((
Em cần gấp :3

Đề sai ,,vì $2^{10}+5^{12}=244141649$ là số nguyên tố :3




#669532 MỘT SỐ PHƯƠNG PHÁP GIẢI TOÁN TỔ HỢP THCS

Posted by viet9a14124869 on 23-01-2017 - 12:40 in Toán rời rạc

 

Đếm số hình tam giác:

attachicon.gifunnamed.png

có 32 tam giác

 

 

bài hay đây:

với x, y, z là các số dương CM

x^2(x+y-z)+y^2(y+z-x)+z^2(x+y-z)bé hơn hoặc bằng 3xyz

 

 

bài này bạn áp dụng bdt hoán vị cho 2 bộ đơn điệu sau $(x,y,z)$ và $(x^2,y^2,z^2)$

 




#669907 MỘT SỐ PHƯƠNG PHÁP GIẢI TOÁN TỔ HỢP THCS

Posted by viet9a14124869 on 25-01-2017 - 21:10 in Toán rời rạc

Bài 23: Cho $a_{1},a_{2},a_{3},................a_{n}$ là 1 hoán vị của 1, 2, 3,............n với n là số lẻ. CMR : $\left ( a_{1}-1 \right )\left ( a_{2} -2\right ).............\left ( a_{n} -n\right )$ là số chẵn . Cho em hỏi hoán vị nghĩa là sao ạ?

nghĩa là các số từ a1 đến an là các số từ 1 đến n nhưng trât tự thay đổi

Vì $(a_{1}-1)+(a_{2}-2)+...+(a_{n}-n)=0$ nên tồn tại ít nhất một hiệu chẵn suy ra q.e.d




#671450 MỘT SỐ PHƯƠNG PHÁP GIẢI TOÁN TỔ HỢP THCS

Posted by viet9a14124869 on 13-02-2017 - 12:52 in Toán rời rạc

không

mình lại nghĩ là có ,,,nhưng chỉ đoán qua thôi ,,,vì số bóng đèn chia hết cho 2 ^-^




#671824 MỘT SỐ PHƯƠNG PHÁP GIẢI TOÁN TỔ HỢP THCS

Posted by viet9a14124869 on 16-02-2017 - 20:40 in Toán rời rạc

Có thể tìm a;b;c là số nguyên nếu a^3+b^3=c^3? Tại sao thế?
Tớ đóng góp vậy đó, xin được giúp đỡ

Ta chọn a=-b và c=0 là xong




#670739 Diễn đàn đã hoạt động trở lại

Posted by viet9a14124869 on 08-02-2017 - 19:49 in Thông báo tổng quan

vậy có cách nào cải thiện tình huống của bạn canhhoang30011999 ko ,,, em dùng firefox cũng bị như vậy ạ 




#670744 Diễn đàn đã hoạt động trở lại

Posted by viet9a14124869 on 08-02-2017 - 20:04 in Thông báo tổng quan

dạ thôi em vào dc rồi anh ạ ^-^




#670743 Diễn đàn đã hoạt động trở lại

Posted by viet9a14124869 on 08-02-2017 - 20:02 in Thông báo tổng quan

em thử bỏ rồi nhưng khi gõ enter là lại có chữ s nữa anh ạ 




#672093 Diễn đàn đã hoạt động trở lại

Posted by viet9a14124869 on 19-02-2017 - 16:15 in Thông báo tổng quan

Cho em hỏi tại sao máy tính của em ko thể kết nối vs diễn đàn vậy ạ?

thế bạn đăng câu này kiểu gì ,,,,




#680284 Topic ôn thi hình học vào cấp 3 chuyên

Posted by viet9a14124869 on 11-05-2017 - 13:38 in Hình học

                                                                                          :D  " Góp vui cho topic "  :D 

Mình xin tham gia topic và mở đầu bằng hai bài toán .

Bài 89 ( APMO 2000 ): Cho tam giác ABC với trung tuyến AM và phân giác AN . Đường thẳng vuông góc với AN tại N cắt AB và AM lần lượt tại P và Q . Đường thẳng vuông góc với AB tại P cắt AN tại O . Chứng minh rằng OQ vuông góc với BC . 

Bài 90 ( sưu tầm ) : Cho tam giác ABC có I là trung điểm BC , đường thẳng d đi qua I cắt AB , AC lần lượt tại M và N , đường thẳng d' đi qua I cắt AB ,AC lần lượt tại Q và P ( M và P nàm cùng phía với BC ) . MP , NQ cắt BC tại E và F . Chứng minh rằng IE = IF .